Tallteori

Her kan brukere av forum utfordre hverandre med morsomme oppgaver og nøtter man ønsker å dele med andre. Dette er altså ikke et sted for desperate skrik om hjelp, de kan man poste i de andre forumene, men et sted for problemløsing på tvers av trinn og fag.

Moderatorer: Vektormannen, espen180, Aleks855, Solar Plexsus, Gustav, Nebuchadnezzar, Janhaa

Svar
Gustav
Tyrann
Tyrann
Innlegg: 4560
Registrert: 12/12-2008 12:44

Bestem alle ordnede par av positive heltall $(n,m)$ slik at $\frac{n^3+1}{mn-1}$ er et heltall.
mrcreosote
Guru
Guru
Innlegg: 1995
Registrert: 10/10-2006 20:58

Del alle par av 2 positive heltall inn i 5 disjunkte mengder som under. Hvis et par ligger i flere av mengdene, fjerner vi det fra alle unntatt den første mengden det ligger i.

i) m,n<6
ii) m=n
iii) m=n+1
iv) n=m+1
v) |m-n|>1

Ved sjekk har i) de 9 løsningene (1,2), (1,3), (2,1), (2,2), (2,5), (3,1), (3,5), (5,2), (5,3). Mengdene ii)-v) har ingen løsninger:

ii) m=n gir at uttrykket kan skrives [tex]n-2+\frac2{n+1}[/tex] slik at vi må ha n=1 for å få et heltall.

iii) m=n+1 gir at uttrykket kan skrives [tex]n-1+\frac{2n-2}{n^2+n-1}[/tex] slik at vi må ha n=1 for å få et heltall.

iv) m=n-1 gir at uttrykket kan skrives [tex]n-1+\frac{2n}{n^2-n-1}[/tex] slik at vi må ha n=1 eller n=2 for å få et heltall.

For v) må vi regne litt mer: [tex]A=\frac{n^3+1}{nm-1}=\frac{n(n^2+m)}{nm-1}-1[/tex], og siden [tex]\gcd(n,nm-1)=1[/tex] må vi ha at [tex]nm-1[/tex] deler [tex]n^2+m[/tex]. Spesielt er [tex]nm-1\le n^2+m[/tex].

Vi kan også skrive [tex]A=\frac{n^2(n+m^2)}{nm-1}-nm-1[/tex], så likeledes er [tex]nm-1\le m^2+n[/tex].

Dette kan vi også skrive som [tex]n\le\frac{m^2+1}{m-1}=m+1+\frac2{m-1}[/tex] og tilsvarende hvor n og m bytter roller, så [tex]2\le|m-n|\le1+\frac2{k-1}[/tex] hvor k er den minste av m og n, og følgelig høyst lik 3. Ved å sette inn n=1, 2 og 3 i [tex]nm-1\le n^2+m[/tex] ser vi at vi ikke får noen nye løsninger, og tilsvarende heller ingen løsninger hvis k=m. (For m=1 får vi [tex]A=\frac{n^2(n+1)}{n-1}-n-1[/tex], så n-1 må dele n+1 som medfører n<4.)

Dette blei ganske langt og krøkkete. Har du en bedre løsning, plutarco? En kan bemerke at (n,m) er en løsning hvis og bare hvis (m,n) er en løsning, men jeg har ikke et direkte argument for dette.
Svar